How can I take double ResRes at a singularity in the residue theorem?

  • Thread starter Thread starter mrsaturn
  • Start date Start date
Click For Summary
The discussion centers on applying the residue theorem to calculate double residues at a singularity where z1 equals z2. A specific function, g = 1/z1 - 1/z2 + 1/(z1 - z2), is presented as an example to clarify the problem. Participants seek guidance on how to compute double residues in this context, particularly in the case of overlapping singularities. The conversation emphasizes the need for a clear method to handle such singularities when using the residue theorem. Understanding this approach is crucial for effectively applying the theorem in complex analysis.
mrsaturn
Messages
2
Reaction score
0
Homework Statement
Let ##g(z_1,z_2)## be a rational function where the only possible singularities are at ##z_1 = 0##, ##z_2 = 0##, and ##z_1 = z_2##. Verify the following: ##\text{Res}_{z_2 = 0}\text{Res}_{z_1 = z_2} g(z_1,z_2) =- \text{Res}_{z_1 = 0}\text{Res}_{z_2 = z_1} g(z_1,z_2)##
Relevant Equations
##\text{Res}_{z_2 = 0}\text{Res}_{z_1 = z_2} g(z_1,z_2) =- \text{Res}_{z_1 = 0}\text{Res}_{z_2 = z_1} g(z_1,z_2)##
I think we should be able to verify this by the residue theorem, but I'm having trouble applying it to the case when there is a singularity at ##z_1 = z_2##
 
Last edited by a moderator:
Physics news on Phys.org
To understand the problem exactly you mean for an example
g=\frac{1}{z_1}-\frac{1}{z_2}+\frac{1}{z_1-z_2}?
 
  • Like
Likes FactChecker
anuttarasammyak said:
To understand the problem exactly you mean for an example
g=\frac{1}{z_1}-\frac{1}{z_2}+\frac{1}{z_1-z_2}?
Yeah exactly
 
Thanks. Then I would like to know how shall I take double ResRes as you do in your formula. Could you show me how to do in this example?
 
Question: A clock's minute hand has length 4 and its hour hand has length 3. What is the distance between the tips at the moment when it is increasing most rapidly?(Putnam Exam Question) Answer: Making assumption that both the hands moves at constant angular velocities, the answer is ## \sqrt{7} .## But don't you think this assumption is somewhat doubtful and wrong?

Similar threads

  • · Replies 3 ·
Replies
3
Views
1K
  • · Replies 6 ·
Replies
6
Views
2K
  • · Replies 2 ·
Replies
2
Views
2K
  • · Replies 14 ·
Replies
14
Views
3K
  • · Replies 5 ·
Replies
5
Views
1K
  • · Replies 2 ·
Replies
2
Views
2K
Replies
6
Views
2K
  • · Replies 8 ·
Replies
8
Views
2K
  • · Replies 1 ·
Replies
1
Views
2K
Replies
4
Views
2K